Which of the following has contributed to a recent banking crisis ?a. Sub prime mortgages
b. wildcat banks
c. Bank fraud
d. Greenbacks

Answers

Answer 1
Answer: The correct answer for the question that is being presented above is this one: "a. Sub prime mortgages" Sub prime mortgages have contributed to a recent banking crisis. Subprime mortgages refers to loans that is usually being offered to borrowers with impaired credit cards.

Related Questions

When countries create tariffs, they __________.a. remove restrictions on imports b. place limits on the amount of goods that can be imported c. set taxes on imported goods d. create common currencies
Who controls the government and juntaa: monarch b: the peoplec: militaryd: president ​
Hat was a major cause of the us recession that began in 2008?
The bilingual education act of 1974 provided: A) that all children should learn a second language B) funds to meet the language needs of non-English speaking students C) that each public school have at least one bilingual teacher D) none of the above
Example of the principle of the separation of powers?

Which of the following were offered aid by the Marshall Plan?all European countries
all western European countries
South Korea
all eastern European countries
all members of NATO

Answers

It was only an European plan: so while other nations also received help, they were note called" Marshal plan".

In fact, all European countries were offered this aid, however, only western Europe accepted, as the Eastern European countries were forced by the Soviet Union to reject it.

I just took the test and the answer is A


Which of the following interest groups attempts to influence public policy at the international level?a. American Civil Liberties Union
b. Greenpeace
c. National Rifle Association
d. Sierra Club

Answers

The correct answer for the question that is being presented above is this one: "b. Greenpeace." The interest groups attempts to influence public policy at the international level is the Greenpeace. Greenpeace is a group of people-powered movement fighting for a green and peaceful future for our oceans, forests, food, climate and democracy.

Answer:b

Explanation:

Colin does not get good grades on standardized tests and does not seem to be making an effort to improve his grades. He is more interested in martial arts and has earned a black belt in this field. In the context of Gardner's theories, Colin has _____.

Answers

Answer:

A bodily-kinesthetic intelligence modality.

Explanation:

As per the theory of mulitple intelligences, the individual (Colin) has a better bodily-kinesthetic intelligence, as he is an outstanding sports practicitioner, and therefore having a motor skill (both gross and fine).

Answer:

???

Explanation:

need to provide answer choices

Your co-worker, Bill, comes into the office and tells you that he is going to play "hooky" and go golfing, believing that the boss will think he is out calling on clients. When your boss comes in, he asks you where Bill is and you reply, "I saw him in the office earlier this morning, but I haven't seen him lately." What would Kant call this misleading statement?

Answers

Answer:A palter

Explanation: what is a palter?

A palter is a statement that someone makes by omitting crucial information which may confuse the receiver of the message.

This is done when someone can not be straight or be factual because they are afraid of the consequences or because they want to mislead the person.

He is misleading the boss by saying he hasn't seen him and he is plainly lying by finding something that may make him not to look guilt .

This may lead to serious consequences because now he lied to the boss even though he knows the truth .

The first armed conflicts of the revolutionary war

Answers

Lexington and concord
The first conflicts in The Revolutionary War were in Concord and Lexington, New Hampshire.
Hope this helps!

David is choosing between two exercise routines. In Routine #1, he burns calories walking. He then runs at a rate that burns calories per minute. In Routine #2, he burns calories walking. He then runs at a rate that burns calories per minute. For what amounts of time spent running will Routine #1 burn at most as many calories as Routine #2? Use for the number of minutes spent running, and solve your inequality for .

Answers

Answer:

The answer is below

Explanation:

The question is not complete, the complete question is in the form of: David is choosing between two exercise routines. In Routine #1, he burns 20 calories walking. He then runs at a rate that burns 10.5 calories per minute. In Routine #2, he burns calories 38 walking. He then runs at a rate that burns  8.5 calories per minute. For what amounts of time spent running will Routine #1 burn at most as many calories as Routine #2? Use for the number of minutes spent running, and solve your inequality for .

Answer:

Let us assume that the number of minutes spent running is t minute. The equation that represents the total calories burnt for routine 1 is given as:

20 + 10.5t

While the total calories burnt for routine 2 is given as:

38 + 8.5t

Since Routine #1 burn at most as many calories as Routine #2, hence it can be represented by the inequality

20 + 10.5t < 38 + 8.5t

Solving the inequality:

10.5t - 8.5t < 38 - 20

2t < 18

t < 9 minutes

For routine 1 to burn at most as many calories as routine 2, the time spent running must be less than 9 minutes

Final answer:

David should run for less than or equal to (W2 - W1) / (R1 - R2) minutes for the calories burned in Routine #1 to be at most equal to that of Routine #2. W1, W2, R1, and R2 represent the number of calories burned walking and running rate in each routine, respectively.

Explanation:

The question doesn't provide specific figures for the amount of calories burned through walking or running in either routine. Therefore, for our purposes, let's denote the number of calories burned walking in Routine #1 and #2 as W1 and W2 respectively, and the rate of calories burned per minute running as R1 and R2 respectively.

If x represents the time (in minutes) David spends running, the total number of calories burned in Routine #1 would be W1 + R1*x, and for Routine #2 it would be W2 + R2*x.

David would burn at most as many calories with Routine #1 as he would with Routine #2 when W1 + R1*x ≤ W2 + R2*x. To solve this inequality for x, you would perform the following steps:

  1. Subtract W1 from both sides of the inequality to isolate the variable terms on one side: R1*x ≤ W2 - W1 + R2*x
  2. Subtract R2*x from both sides: (R1 - R2) * x ≤ W2 - W1
  3. Divide by R1 - R2 to solve for x: x ≤ (W2 - W1) / (R1 - R2)
  4. The amount of time spent running for which Routine #1 burns at most as many calories as Routine #2 is given by x ≤ (W2 - W1) / (R1 - R2).

Learn more about Inequalities here:

brainly.com/question/32625151

#SPJ3